An inequality involving multi-index












3












$begingroup$


I came across these inequalities while learning about Schwartz functions (Classical Fourier Analysis, Grafakos) and I have no idea how to prove this:



For $x in mathbb{R}^{n}$ and $alpha = (alpha_{1}, ldots, alpha_{n}) in mathbb{N}^{n}$, we set



$$ x^{alpha} = x_{1}^{alpha_{1}}cdots x_{n}^{alpha_{n}}.$$



Then prove that there exists a constant $c_{n,alpha}$ such that




$$left| x^{alpha}right| leq c_{n,alpha}|x|^{|alpha|}$$




where $|alpha| = alpha_{1} + cdots + alpha_{n}$.



Conversely, for every $k in mathbb{N}$, there exists a $C_{n,k}$ such that




$$|x|^{k} leq C_{n,k}sumlimits_{|beta| = k}|x^{beta}|$$




Any help would be appreciated.










share|cite|improve this question









$endgroup$

















    3












    $begingroup$


    I came across these inequalities while learning about Schwartz functions (Classical Fourier Analysis, Grafakos) and I have no idea how to prove this:



    For $x in mathbb{R}^{n}$ and $alpha = (alpha_{1}, ldots, alpha_{n}) in mathbb{N}^{n}$, we set



    $$ x^{alpha} = x_{1}^{alpha_{1}}cdots x_{n}^{alpha_{n}}.$$



    Then prove that there exists a constant $c_{n,alpha}$ such that




    $$left| x^{alpha}right| leq c_{n,alpha}|x|^{|alpha|}$$




    where $|alpha| = alpha_{1} + cdots + alpha_{n}$.



    Conversely, for every $k in mathbb{N}$, there exists a $C_{n,k}$ such that




    $$|x|^{k} leq C_{n,k}sumlimits_{|beta| = k}|x^{beta}|$$




    Any help would be appreciated.










    share|cite|improve this question









    $endgroup$















      3












      3








      3


      1



      $begingroup$


      I came across these inequalities while learning about Schwartz functions (Classical Fourier Analysis, Grafakos) and I have no idea how to prove this:



      For $x in mathbb{R}^{n}$ and $alpha = (alpha_{1}, ldots, alpha_{n}) in mathbb{N}^{n}$, we set



      $$ x^{alpha} = x_{1}^{alpha_{1}}cdots x_{n}^{alpha_{n}}.$$



      Then prove that there exists a constant $c_{n,alpha}$ such that




      $$left| x^{alpha}right| leq c_{n,alpha}|x|^{|alpha|}$$




      where $|alpha| = alpha_{1} + cdots + alpha_{n}$.



      Conversely, for every $k in mathbb{N}$, there exists a $C_{n,k}$ such that




      $$|x|^{k} leq C_{n,k}sumlimits_{|beta| = k}|x^{beta}|$$




      Any help would be appreciated.










      share|cite|improve this question









      $endgroup$




      I came across these inequalities while learning about Schwartz functions (Classical Fourier Analysis, Grafakos) and I have no idea how to prove this:



      For $x in mathbb{R}^{n}$ and $alpha = (alpha_{1}, ldots, alpha_{n}) in mathbb{N}^{n}$, we set



      $$ x^{alpha} = x_{1}^{alpha_{1}}cdots x_{n}^{alpha_{n}}.$$



      Then prove that there exists a constant $c_{n,alpha}$ such that




      $$left| x^{alpha}right| leq c_{n,alpha}|x|^{|alpha|}$$




      where $|alpha| = alpha_{1} + cdots + alpha_{n}$.



      Conversely, for every $k in mathbb{N}$, there exists a $C_{n,k}$ such that




      $$|x|^{k} leq C_{n,k}sumlimits_{|beta| = k}|x^{beta}|$$




      Any help would be appreciated.







      real-analysis multivariable-calculus inequality






      share|cite|improve this question













      share|cite|improve this question











      share|cite|improve this question




      share|cite|improve this question










      asked Jul 26 '13 at 6:24









      Vishal GuptaVishal Gupta

      4,64521843




      4,64521843






















          1 Answer
          1






          active

          oldest

          votes


















          3












          $begingroup$

          The first inequality is clear at $x=0$ with any constant . Now note that it suffices to prove it for $|x|=1$. This is because if it is true on the unit sphere then since $xneq0$ then $y=(frac{x_{1}}{|x|},frac{x_{2}}{|x|},...,frac{x_{n}}{|x|})=frac{x}{|x|}in S^{n-1}$ and then we have



          $$begin{align} frac{|x^{alpha}|}{|x|^{|alpha|}}
          & = frac{ sqrt{ x_1^{2alpha_1} + dots + x_n^{2alpha_n} } }{|x|^{|alpha|} } \
          & = bigg( frac{ x_1^{2alpha_1} + dots + x_n^{2alpha_n} }{|x|^{2|alpha|}} bigg)^{frac{1}{2}}
          \
          & = bigg( frac{x_1^{2alpha_1}}{|x|^{2|alpha|}} + dots + frac{x_n^{2alpha_n}}{|x|^{2|alpha|}} bigg)^{frac{1}{2}}
          \
          & leq bigg[ bigg(frac{x_1}{|x|}bigg)^{2alpha_1} + dots + bigg(frac{x_n}{|x|}bigg)^{2alpha_n}bigg]^{frac{1}{2}}
          \
          & = bigg| bigg( frac{x_1^{alpha_1}}{|x|^{alpha_1}}, dots, frac{x_n^{alpha_n}}{|x|^{alpha_n}} bigg) bigg|
          \
          & = | y^{alpha} |
          \
          & leq c_{n,alpha} |y|^{|alpha|}
          \
          & = c_{n,alpha}bigg|bigg(frac{x_{1}}{|x|},frac{x_{2}}{|x|},...frac{x_{n}}{|x|}bigg)bigg|^{|alpha|}
          \
          & = c_{n,alpha}frac{1}{|x|^{|alpha|}}|(x_{1},x_{2},...x_{n})|^{|alpha|}
          \
          & = c_{n,alpha}frac{|x|^{|alpha|}}{|x|^{|alpha|}}
          = c_{n,alpha}
          end{align}$$



          So $|x^{alpha}|le c_{n,alpha}|x|^{|alpha|}$.



          Now we prove this inequality on the unit sphere.



          $vert x^{alpha}vert=|x_1^{alpha_1}||x_2^{alpha_{2}}|...|x_n^{alpha_n}|lefrac{1}{n}sum_{k=1}^{n}|x_{k}|^{nalpha_{k}}lefrac{1}{n}big(sum_{k=1}^{n}|x_{k}|^{alpha_{k}}big)^{n}$



          $lefrac{1}{n}big(sum_{k=1}^{n}(1+|x_{k}|)^{alpha_{k}}big)^{n}lefrac{1}{n}big(sum_{k=1}^{n}(1+|x|)^{alpha_k}big)^{n}lefrac{1}{n}big(n(1+|x|)^{|alpha|})^{n}=n^{n-1}2^{n|alpha|}$.



          Note that I have used the Arithmetic Geometric Inequality to get the first inequality in the above proof. The second inequality follows from the inequality: $(x_{1}+x_{2}+...+x_{n})^{k}=sum_{alpha_{1}+...+alpha_{n}=k}binom{k}{alpha_{1}, alpha_{2},...,alpha_{n}}x^{alpha_{1}}...x^{alpha_{n}}ge x_{1}^{k}+...+x_{n}^{k}$ which is true when $x_{1},...x_{n}$ are all non-negative. Also note that $|x_{k}|le1+|x_{k}|$ and $|x_{k}|le sqrt{x_{1}^{2}+...+x_{n}^{2}}=|x|$ for all $k=1,2,...,n$.



          For the second inequality we attack as follows:



          $|x|^{k}=big(sum_{k=1}^{n}|x_{k}|^{2}big)^{frac{k}{2}}lebig(sum_{k=1}^{n}|x_{k}|big)^{frac{2k}{2}}=big(sum_{k=1}^{n}|x_{k}|big)^{k}=sum_{|beta|=k}frac{k!}{beta!}|x^{beta}|lebig(sum_{|beta|=k}big(frac{k!}{beta!}big)big)big(sum_{|beta|=k}|x^{beta}|big)=n^{k}big(sum_{|beta|=k}|x^{beta}|big)$.






          share|cite|improve this answer











          $endgroup$













            Your Answer





            StackExchange.ifUsing("editor", function () {
            return StackExchange.using("mathjaxEditing", function () {
            StackExchange.MarkdownEditor.creationCallbacks.add(function (editor, postfix) {
            StackExchange.mathjaxEditing.prepareWmdForMathJax(editor, postfix, [["$", "$"], ["\\(","\\)"]]);
            });
            });
            }, "mathjax-editing");

            StackExchange.ready(function() {
            var channelOptions = {
            tags: "".split(" "),
            id: "69"
            };
            initTagRenderer("".split(" "), "".split(" "), channelOptions);

            StackExchange.using("externalEditor", function() {
            // Have to fire editor after snippets, if snippets enabled
            if (StackExchange.settings.snippets.snippetsEnabled) {
            StackExchange.using("snippets", function() {
            createEditor();
            });
            }
            else {
            createEditor();
            }
            });

            function createEditor() {
            StackExchange.prepareEditor({
            heartbeatType: 'answer',
            autoActivateHeartbeat: false,
            convertImagesToLinks: true,
            noModals: true,
            showLowRepImageUploadWarning: true,
            reputationToPostImages: 10,
            bindNavPrevention: true,
            postfix: "",
            imageUploader: {
            brandingHtml: "Powered by u003ca class="icon-imgur-white" href="https://imgur.com/"u003eu003c/au003e",
            contentPolicyHtml: "User contributions licensed under u003ca href="https://creativecommons.org/licenses/by-sa/3.0/"u003ecc by-sa 3.0 with attribution requiredu003c/au003e u003ca href="https://stackoverflow.com/legal/content-policy"u003e(content policy)u003c/au003e",
            allowUrls: true
            },
            noCode: true, onDemand: true,
            discardSelector: ".discard-answer"
            ,immediatelyShowMarkdownHelp:true
            });


            }
            });














            draft saved

            draft discarded


















            StackExchange.ready(
            function () {
            StackExchange.openid.initPostLogin('.new-post-login', 'https%3a%2f%2fmath.stackexchange.com%2fquestions%2f452524%2fan-inequality-involving-multi-index%23new-answer', 'question_page');
            }
            );

            Post as a guest















            Required, but never shown

























            1 Answer
            1






            active

            oldest

            votes








            1 Answer
            1






            active

            oldest

            votes









            active

            oldest

            votes






            active

            oldest

            votes









            3












            $begingroup$

            The first inequality is clear at $x=0$ with any constant . Now note that it suffices to prove it for $|x|=1$. This is because if it is true on the unit sphere then since $xneq0$ then $y=(frac{x_{1}}{|x|},frac{x_{2}}{|x|},...,frac{x_{n}}{|x|})=frac{x}{|x|}in S^{n-1}$ and then we have



            $$begin{align} frac{|x^{alpha}|}{|x|^{|alpha|}}
            & = frac{ sqrt{ x_1^{2alpha_1} + dots + x_n^{2alpha_n} } }{|x|^{|alpha|} } \
            & = bigg( frac{ x_1^{2alpha_1} + dots + x_n^{2alpha_n} }{|x|^{2|alpha|}} bigg)^{frac{1}{2}}
            \
            & = bigg( frac{x_1^{2alpha_1}}{|x|^{2|alpha|}} + dots + frac{x_n^{2alpha_n}}{|x|^{2|alpha|}} bigg)^{frac{1}{2}}
            \
            & leq bigg[ bigg(frac{x_1}{|x|}bigg)^{2alpha_1} + dots + bigg(frac{x_n}{|x|}bigg)^{2alpha_n}bigg]^{frac{1}{2}}
            \
            & = bigg| bigg( frac{x_1^{alpha_1}}{|x|^{alpha_1}}, dots, frac{x_n^{alpha_n}}{|x|^{alpha_n}} bigg) bigg|
            \
            & = | y^{alpha} |
            \
            & leq c_{n,alpha} |y|^{|alpha|}
            \
            & = c_{n,alpha}bigg|bigg(frac{x_{1}}{|x|},frac{x_{2}}{|x|},...frac{x_{n}}{|x|}bigg)bigg|^{|alpha|}
            \
            & = c_{n,alpha}frac{1}{|x|^{|alpha|}}|(x_{1},x_{2},...x_{n})|^{|alpha|}
            \
            & = c_{n,alpha}frac{|x|^{|alpha|}}{|x|^{|alpha|}}
            = c_{n,alpha}
            end{align}$$



            So $|x^{alpha}|le c_{n,alpha}|x|^{|alpha|}$.



            Now we prove this inequality on the unit sphere.



            $vert x^{alpha}vert=|x_1^{alpha_1}||x_2^{alpha_{2}}|...|x_n^{alpha_n}|lefrac{1}{n}sum_{k=1}^{n}|x_{k}|^{nalpha_{k}}lefrac{1}{n}big(sum_{k=1}^{n}|x_{k}|^{alpha_{k}}big)^{n}$



            $lefrac{1}{n}big(sum_{k=1}^{n}(1+|x_{k}|)^{alpha_{k}}big)^{n}lefrac{1}{n}big(sum_{k=1}^{n}(1+|x|)^{alpha_k}big)^{n}lefrac{1}{n}big(n(1+|x|)^{|alpha|})^{n}=n^{n-1}2^{n|alpha|}$.



            Note that I have used the Arithmetic Geometric Inequality to get the first inequality in the above proof. The second inequality follows from the inequality: $(x_{1}+x_{2}+...+x_{n})^{k}=sum_{alpha_{1}+...+alpha_{n}=k}binom{k}{alpha_{1}, alpha_{2},...,alpha_{n}}x^{alpha_{1}}...x^{alpha_{n}}ge x_{1}^{k}+...+x_{n}^{k}$ which is true when $x_{1},...x_{n}$ are all non-negative. Also note that $|x_{k}|le1+|x_{k}|$ and $|x_{k}|le sqrt{x_{1}^{2}+...+x_{n}^{2}}=|x|$ for all $k=1,2,...,n$.



            For the second inequality we attack as follows:



            $|x|^{k}=big(sum_{k=1}^{n}|x_{k}|^{2}big)^{frac{k}{2}}lebig(sum_{k=1}^{n}|x_{k}|big)^{frac{2k}{2}}=big(sum_{k=1}^{n}|x_{k}|big)^{k}=sum_{|beta|=k}frac{k!}{beta!}|x^{beta}|lebig(sum_{|beta|=k}big(frac{k!}{beta!}big)big)big(sum_{|beta|=k}|x^{beta}|big)=n^{k}big(sum_{|beta|=k}|x^{beta}|big)$.






            share|cite|improve this answer











            $endgroup$


















              3












              $begingroup$

              The first inequality is clear at $x=0$ with any constant . Now note that it suffices to prove it for $|x|=1$. This is because if it is true on the unit sphere then since $xneq0$ then $y=(frac{x_{1}}{|x|},frac{x_{2}}{|x|},...,frac{x_{n}}{|x|})=frac{x}{|x|}in S^{n-1}$ and then we have



              $$begin{align} frac{|x^{alpha}|}{|x|^{|alpha|}}
              & = frac{ sqrt{ x_1^{2alpha_1} + dots + x_n^{2alpha_n} } }{|x|^{|alpha|} } \
              & = bigg( frac{ x_1^{2alpha_1} + dots + x_n^{2alpha_n} }{|x|^{2|alpha|}} bigg)^{frac{1}{2}}
              \
              & = bigg( frac{x_1^{2alpha_1}}{|x|^{2|alpha|}} + dots + frac{x_n^{2alpha_n}}{|x|^{2|alpha|}} bigg)^{frac{1}{2}}
              \
              & leq bigg[ bigg(frac{x_1}{|x|}bigg)^{2alpha_1} + dots + bigg(frac{x_n}{|x|}bigg)^{2alpha_n}bigg]^{frac{1}{2}}
              \
              & = bigg| bigg( frac{x_1^{alpha_1}}{|x|^{alpha_1}}, dots, frac{x_n^{alpha_n}}{|x|^{alpha_n}} bigg) bigg|
              \
              & = | y^{alpha} |
              \
              & leq c_{n,alpha} |y|^{|alpha|}
              \
              & = c_{n,alpha}bigg|bigg(frac{x_{1}}{|x|},frac{x_{2}}{|x|},...frac{x_{n}}{|x|}bigg)bigg|^{|alpha|}
              \
              & = c_{n,alpha}frac{1}{|x|^{|alpha|}}|(x_{1},x_{2},...x_{n})|^{|alpha|}
              \
              & = c_{n,alpha}frac{|x|^{|alpha|}}{|x|^{|alpha|}}
              = c_{n,alpha}
              end{align}$$



              So $|x^{alpha}|le c_{n,alpha}|x|^{|alpha|}$.



              Now we prove this inequality on the unit sphere.



              $vert x^{alpha}vert=|x_1^{alpha_1}||x_2^{alpha_{2}}|...|x_n^{alpha_n}|lefrac{1}{n}sum_{k=1}^{n}|x_{k}|^{nalpha_{k}}lefrac{1}{n}big(sum_{k=1}^{n}|x_{k}|^{alpha_{k}}big)^{n}$



              $lefrac{1}{n}big(sum_{k=1}^{n}(1+|x_{k}|)^{alpha_{k}}big)^{n}lefrac{1}{n}big(sum_{k=1}^{n}(1+|x|)^{alpha_k}big)^{n}lefrac{1}{n}big(n(1+|x|)^{|alpha|})^{n}=n^{n-1}2^{n|alpha|}$.



              Note that I have used the Arithmetic Geometric Inequality to get the first inequality in the above proof. The second inequality follows from the inequality: $(x_{1}+x_{2}+...+x_{n})^{k}=sum_{alpha_{1}+...+alpha_{n}=k}binom{k}{alpha_{1}, alpha_{2},...,alpha_{n}}x^{alpha_{1}}...x^{alpha_{n}}ge x_{1}^{k}+...+x_{n}^{k}$ which is true when $x_{1},...x_{n}$ are all non-negative. Also note that $|x_{k}|le1+|x_{k}|$ and $|x_{k}|le sqrt{x_{1}^{2}+...+x_{n}^{2}}=|x|$ for all $k=1,2,...,n$.



              For the second inequality we attack as follows:



              $|x|^{k}=big(sum_{k=1}^{n}|x_{k}|^{2}big)^{frac{k}{2}}lebig(sum_{k=1}^{n}|x_{k}|big)^{frac{2k}{2}}=big(sum_{k=1}^{n}|x_{k}|big)^{k}=sum_{|beta|=k}frac{k!}{beta!}|x^{beta}|lebig(sum_{|beta|=k}big(frac{k!}{beta!}big)big)big(sum_{|beta|=k}|x^{beta}|big)=n^{k}big(sum_{|beta|=k}|x^{beta}|big)$.






              share|cite|improve this answer











              $endgroup$
















                3












                3








                3





                $begingroup$

                The first inequality is clear at $x=0$ with any constant . Now note that it suffices to prove it for $|x|=1$. This is because if it is true on the unit sphere then since $xneq0$ then $y=(frac{x_{1}}{|x|},frac{x_{2}}{|x|},...,frac{x_{n}}{|x|})=frac{x}{|x|}in S^{n-1}$ and then we have



                $$begin{align} frac{|x^{alpha}|}{|x|^{|alpha|}}
                & = frac{ sqrt{ x_1^{2alpha_1} + dots + x_n^{2alpha_n} } }{|x|^{|alpha|} } \
                & = bigg( frac{ x_1^{2alpha_1} + dots + x_n^{2alpha_n} }{|x|^{2|alpha|}} bigg)^{frac{1}{2}}
                \
                & = bigg( frac{x_1^{2alpha_1}}{|x|^{2|alpha|}} + dots + frac{x_n^{2alpha_n}}{|x|^{2|alpha|}} bigg)^{frac{1}{2}}
                \
                & leq bigg[ bigg(frac{x_1}{|x|}bigg)^{2alpha_1} + dots + bigg(frac{x_n}{|x|}bigg)^{2alpha_n}bigg]^{frac{1}{2}}
                \
                & = bigg| bigg( frac{x_1^{alpha_1}}{|x|^{alpha_1}}, dots, frac{x_n^{alpha_n}}{|x|^{alpha_n}} bigg) bigg|
                \
                & = | y^{alpha} |
                \
                & leq c_{n,alpha} |y|^{|alpha|}
                \
                & = c_{n,alpha}bigg|bigg(frac{x_{1}}{|x|},frac{x_{2}}{|x|},...frac{x_{n}}{|x|}bigg)bigg|^{|alpha|}
                \
                & = c_{n,alpha}frac{1}{|x|^{|alpha|}}|(x_{1},x_{2},...x_{n})|^{|alpha|}
                \
                & = c_{n,alpha}frac{|x|^{|alpha|}}{|x|^{|alpha|}}
                = c_{n,alpha}
                end{align}$$



                So $|x^{alpha}|le c_{n,alpha}|x|^{|alpha|}$.



                Now we prove this inequality on the unit sphere.



                $vert x^{alpha}vert=|x_1^{alpha_1}||x_2^{alpha_{2}}|...|x_n^{alpha_n}|lefrac{1}{n}sum_{k=1}^{n}|x_{k}|^{nalpha_{k}}lefrac{1}{n}big(sum_{k=1}^{n}|x_{k}|^{alpha_{k}}big)^{n}$



                $lefrac{1}{n}big(sum_{k=1}^{n}(1+|x_{k}|)^{alpha_{k}}big)^{n}lefrac{1}{n}big(sum_{k=1}^{n}(1+|x|)^{alpha_k}big)^{n}lefrac{1}{n}big(n(1+|x|)^{|alpha|})^{n}=n^{n-1}2^{n|alpha|}$.



                Note that I have used the Arithmetic Geometric Inequality to get the first inequality in the above proof. The second inequality follows from the inequality: $(x_{1}+x_{2}+...+x_{n})^{k}=sum_{alpha_{1}+...+alpha_{n}=k}binom{k}{alpha_{1}, alpha_{2},...,alpha_{n}}x^{alpha_{1}}...x^{alpha_{n}}ge x_{1}^{k}+...+x_{n}^{k}$ which is true when $x_{1},...x_{n}$ are all non-negative. Also note that $|x_{k}|le1+|x_{k}|$ and $|x_{k}|le sqrt{x_{1}^{2}+...+x_{n}^{2}}=|x|$ for all $k=1,2,...,n$.



                For the second inequality we attack as follows:



                $|x|^{k}=big(sum_{k=1}^{n}|x_{k}|^{2}big)^{frac{k}{2}}lebig(sum_{k=1}^{n}|x_{k}|big)^{frac{2k}{2}}=big(sum_{k=1}^{n}|x_{k}|big)^{k}=sum_{|beta|=k}frac{k!}{beta!}|x^{beta}|lebig(sum_{|beta|=k}big(frac{k!}{beta!}big)big)big(sum_{|beta|=k}|x^{beta}|big)=n^{k}big(sum_{|beta|=k}|x^{beta}|big)$.






                share|cite|improve this answer











                $endgroup$



                The first inequality is clear at $x=0$ with any constant . Now note that it suffices to prove it for $|x|=1$. This is because if it is true on the unit sphere then since $xneq0$ then $y=(frac{x_{1}}{|x|},frac{x_{2}}{|x|},...,frac{x_{n}}{|x|})=frac{x}{|x|}in S^{n-1}$ and then we have



                $$begin{align} frac{|x^{alpha}|}{|x|^{|alpha|}}
                & = frac{ sqrt{ x_1^{2alpha_1} + dots + x_n^{2alpha_n} } }{|x|^{|alpha|} } \
                & = bigg( frac{ x_1^{2alpha_1} + dots + x_n^{2alpha_n} }{|x|^{2|alpha|}} bigg)^{frac{1}{2}}
                \
                & = bigg( frac{x_1^{2alpha_1}}{|x|^{2|alpha|}} + dots + frac{x_n^{2alpha_n}}{|x|^{2|alpha|}} bigg)^{frac{1}{2}}
                \
                & leq bigg[ bigg(frac{x_1}{|x|}bigg)^{2alpha_1} + dots + bigg(frac{x_n}{|x|}bigg)^{2alpha_n}bigg]^{frac{1}{2}}
                \
                & = bigg| bigg( frac{x_1^{alpha_1}}{|x|^{alpha_1}}, dots, frac{x_n^{alpha_n}}{|x|^{alpha_n}} bigg) bigg|
                \
                & = | y^{alpha} |
                \
                & leq c_{n,alpha} |y|^{|alpha|}
                \
                & = c_{n,alpha}bigg|bigg(frac{x_{1}}{|x|},frac{x_{2}}{|x|},...frac{x_{n}}{|x|}bigg)bigg|^{|alpha|}
                \
                & = c_{n,alpha}frac{1}{|x|^{|alpha|}}|(x_{1},x_{2},...x_{n})|^{|alpha|}
                \
                & = c_{n,alpha}frac{|x|^{|alpha|}}{|x|^{|alpha|}}
                = c_{n,alpha}
                end{align}$$



                So $|x^{alpha}|le c_{n,alpha}|x|^{|alpha|}$.



                Now we prove this inequality on the unit sphere.



                $vert x^{alpha}vert=|x_1^{alpha_1}||x_2^{alpha_{2}}|...|x_n^{alpha_n}|lefrac{1}{n}sum_{k=1}^{n}|x_{k}|^{nalpha_{k}}lefrac{1}{n}big(sum_{k=1}^{n}|x_{k}|^{alpha_{k}}big)^{n}$



                $lefrac{1}{n}big(sum_{k=1}^{n}(1+|x_{k}|)^{alpha_{k}}big)^{n}lefrac{1}{n}big(sum_{k=1}^{n}(1+|x|)^{alpha_k}big)^{n}lefrac{1}{n}big(n(1+|x|)^{|alpha|})^{n}=n^{n-1}2^{n|alpha|}$.



                Note that I have used the Arithmetic Geometric Inequality to get the first inequality in the above proof. The second inequality follows from the inequality: $(x_{1}+x_{2}+...+x_{n})^{k}=sum_{alpha_{1}+...+alpha_{n}=k}binom{k}{alpha_{1}, alpha_{2},...,alpha_{n}}x^{alpha_{1}}...x^{alpha_{n}}ge x_{1}^{k}+...+x_{n}^{k}$ which is true when $x_{1},...x_{n}$ are all non-negative. Also note that $|x_{k}|le1+|x_{k}|$ and $|x_{k}|le sqrt{x_{1}^{2}+...+x_{n}^{2}}=|x|$ for all $k=1,2,...,n$.



                For the second inequality we attack as follows:



                $|x|^{k}=big(sum_{k=1}^{n}|x_{k}|^{2}big)^{frac{k}{2}}lebig(sum_{k=1}^{n}|x_{k}|big)^{frac{2k}{2}}=big(sum_{k=1}^{n}|x_{k}|big)^{k}=sum_{|beta|=k}frac{k!}{beta!}|x^{beta}|lebig(sum_{|beta|=k}big(frac{k!}{beta!}big)big)big(sum_{|beta|=k}|x^{beta}|big)=n^{k}big(sum_{|beta|=k}|x^{beta}|big)$.







                share|cite|improve this answer














                share|cite|improve this answer



                share|cite|improve this answer








                edited Dec 26 '18 at 16:30









                Namaste

                1




                1










                answered Jul 29 '13 at 19:54









                user71352user71352

                11.4k21025




                11.4k21025






























                    draft saved

                    draft discarded




















































                    Thanks for contributing an answer to Mathematics Stack Exchange!


                    • Please be sure to answer the question. Provide details and share your research!

                    But avoid



                    • Asking for help, clarification, or responding to other answers.

                    • Making statements based on opinion; back them up with references or personal experience.


                    Use MathJax to format equations. MathJax reference.


                    To learn more, see our tips on writing great answers.




                    draft saved


                    draft discarded














                    StackExchange.ready(
                    function () {
                    StackExchange.openid.initPostLogin('.new-post-login', 'https%3a%2f%2fmath.stackexchange.com%2fquestions%2f452524%2fan-inequality-involving-multi-index%23new-answer', 'question_page');
                    }
                    );

                    Post as a guest















                    Required, but never shown





















































                    Required, but never shown














                    Required, but never shown












                    Required, but never shown







                    Required, but never shown

































                    Required, but never shown














                    Required, but never shown












                    Required, but never shown







                    Required, but never shown







                    Popular posts from this blog

                    How do I know what Microsoft account the skydrive app is syncing to?

                    When does type information flow backwards in C++?

                    Grease: Live!